Petite inégalité qui me casse la tête ...

Olympiades mathématiques, énigmes et défis
Avatar de l’utilisateur
Olympus
Modérateur
Messages: 1668
Enregistré le: 12 Mai 2009, 13:00

Petite inégalité qui me casse la tête ...

par Olympus » 16 Nov 2009, 14:58

Bonjour,

bon voilà il ne me reste que 3 jours pour me préparer aux olympiades de Kénitra/Maroc, et je voudrais à tout prix ne pas être éliminé au premier tour . Donc j'ai comme un no-life pris quelques inégalités d'olympiades trouvées sur le Net, ça va bien jusqu'ici ( 13 exos d'inégalités réalisés sur 16 ), mais je bloque sur celle-ci depuis hier, quelqu'un pourrait-il me donner un petit coup de pouce ?

Soient , et trois réels strictement positifs, et ( pas besoin vu que de toute façon l'inégalité est symétrique mais osef ) .

Montrer que : .

Bon voilà où j'en suis :

cette inégalité ayant une forte ressemblance avec celle-ci , je vais la prouver et peut-être qu'elle servira ( mais j'ai pas encore réussi à créer une connexion entre les deux inégalités ) .

Facile à prouver par réordonnement, mais faisons comme si on ne l'a pas vu en utilisant uniquement le bagage du lycée :

On sait que .

Donc :








L'idée serait d'exploiter cette inégalité pour arriver à celle qu'il fallait démontrer, puisqu'on voit que chaque nombre est multiplié par , et respectivement, mais je n'y arrive pas :cry:

Ou alors, ce n'est pas la bonne piste et je perds du temps là dedans ?

Merci !



laquestion
Membre Naturel
Messages: 93
Enregistré le: 07 Nov 2009, 18:42

par laquestion » 16 Nov 2009, 16:07

pourquoi l'inegalité est symetrique ?

Avatar de l’utilisateur
Olympus
Modérateur
Messages: 1668
Enregistré le: 12 Mai 2009, 13:00

par Olympus » 16 Nov 2009, 18:53

laquestion a écrit:pourquoi l'inegalité est symetrique ?


Euh, parce qu'elle n'implique pas un ordre précis des variables et donc qu'on peut supposer nous même, par symétrie des rôles, que ou vu qu'il y aura de toute façon un plus grand et un plus petit élément, ou alors me suis-je trompé ?

laquestion
Membre Naturel
Messages: 93
Enregistré le: 07 Nov 2009, 18:42

par laquestion » 16 Nov 2009, 20:46

je crois. pourquoi ne pas considerer x^(z/y) plutot que x^(y/z) à ce moment là...

Avatar de l’utilisateur
Olympus
Modérateur
Messages: 1668
Enregistré le: 12 Mai 2009, 13:00

par Olympus » 16 Nov 2009, 22:11

Euh les fractions ne sont pas en exposants ^^

Avatar de l’utilisateur
Olympus
Modérateur
Messages: 1668
Enregistré le: 12 Mai 2009, 13:00

par Olympus » 16 Nov 2009, 22:17

Voilà j'ai un peu changé le code Latex pour que ça n'apparaisse pas en exposant ( même si mon code était correct, mais le rendu pas lisible comme je vois ) .

laquestion
Membre Naturel
Messages: 93
Enregistré le: 07 Nov 2009, 18:42

par laquestion » 17 Nov 2009, 15:56

ok merci bien. heureusement que tu me l'a dit car j'etais entrain de pietiner. je pietine toujours mais avec plus d'espoir. je crois que l'inegalité n'est pas symetrique (la division n'est pas commutative).

laquestion
Membre Naturel
Messages: 93
Enregistré le: 07 Nov 2009, 18:42

par laquestion » 17 Nov 2009, 16:03

si l'inegalité est vraie, elle est valable pour tout rééls tels que |x| superieur à |y| superieur à |z| car ax, ay et az veriefient aussi l'inegalité.(et par densité des rationnels)

laquestion
Membre Naturel
Messages: 93
Enregistré le: 07 Nov 2009, 18:42

par laquestion » 20 Nov 2009, 20:33

test tex je vais l enlever

Avatar de l’utilisateur
Olympus
Modérateur
Messages: 1668
Enregistré le: 12 Mai 2009, 13:00

par Olympus » 21 Nov 2009, 17:11

Je suis toujours intéressé par une solution :-)

Merci !

laquestion
Membre Naturel
Messages: 93
Enregistré le: 07 Nov 2009, 18:42

par laquestion » 21 Nov 2009, 21:42

Olympus a écrit:Je suis toujours intéressé par une solution :-)

Merci !

je crois que j'ai trouvé.
on peut supposer y=1 comme je l'ai dit avant. on pose x=expa et z=exp(-b) a et b positifs. on mets le membre de gauche à droite et on derive la fonction suivant a et on montre sans difficulté qu'elle est croissante. on a plus qu'a verifier que l'inegalité est verifiée pour x=1 et y=1.
je donnerai plus de detail eventuellement...

Avatar de l’utilisateur
Ben314
Le Ben
Messages: 21482
Enregistré le: 11 Nov 2009, 23:53

par Ben314 » 21 Nov 2009, 21:57

L'inéquation de départ n'est pas symétrique en (x,y,z) :
si on remplace (x,y,z) par (y,z,x) cela ne change rien, mais si on fait
(x,y,z)->(y,x,z) il me semble que ce n'est plus la même.

J'ai l'impression d'avoir une solution mais elle est tellemen "bourrine" que je sais pas si j'ose la doner (en plus je risque de m'être gourré), bon, enfin :
Je fait tout passer à gauche et je note f(x) la fonction (en fait, je regarde x,y,z comme des réels tels que x>=y>=z>0 avec y,z fixés une bonne fois pour toute)
1) Je calcule f''(x) => ... f''(x)>0 pour tout x>0 donc f' est croissante sur [0,+infty[.
2) Je calcule f'(y)=...=(y-z)(2y^2-z^2)/(yz)>=0 comme f' est croissante, j'en déduit que f'(x)>=0 pour tout x>=y et donc que f est croissante sur [y,infty[.
3) Je calcule f(y)=y(y-z)^2/z>=0 donc f(x)>=f(y)>=0.

Je sais, c'est trés laid (j'ai honte) et en plus, ca risque d'être faux...
Qui n'entend qu'un son n'entend qu'une sonnerie. Signé : Sonfucius

mito94
Membre Relatif
Messages: 165
Enregistré le: 21 Juin 2009, 10:03

par mito94 » 21 Nov 2009, 22:58

il est possible que tous soit faux c'est la premiere fois que je confronte a ce genre d'exo et ma résolution semble bien simple ! :doh:



on obtient

De plus

et


de plus car
De meme , car

Donc

De plus il faut que

d'ou que

donc


Donc

On obtient donc

donc pour on obtient

laquestion
Membre Naturel
Messages: 93
Enregistré le: 07 Nov 2009, 18:42

par laquestion » 22 Nov 2009, 00:16

mito94 a écrit:

c'est la que ça coince...
(ma "solution" ne vous convient pas ?)

Avatar de l’utilisateur
Olympus
Modérateur
Messages: 1668
Enregistré le: 12 Mai 2009, 13:00

par Olympus » 22 Nov 2009, 00:18

mito94 a écrit:


Peut-être que j'ai pas bien suivi, mais cela ne devrait pas plutôt être x-y >= 0 ? ( même chose pour y-z )

Avatar de l’utilisateur
Olympus
Modérateur
Messages: 1668
Enregistré le: 12 Mai 2009, 13:00

par Olympus » 22 Nov 2009, 00:21

Oups ma faute, x,y et z sont trois réels strictement positifs, je corrige l'énoncé ^^"

PS : euh je n'ai aucune notion en dérivées etc... ( pas encore faits ça en cours ), mais si ça permet de résoudre l'exercice, pourquoi pas ! :we:

Avatar de l’utilisateur
Ben314
Le Ben
Messages: 21482
Enregistré le: 11 Nov 2009, 23:53

par Ben314 » 22 Nov 2009, 00:25

laquestion, en ce qui me concerne ta soluce "me convient" : c'est le même style que ce que j'ai fait (mais je trouve pas cela super beau : l'énoncé était "presque" symétrique et la solution quasiment pas)
Qui n'entend qu'un son n'entend qu'une sonnerie. Signé : Sonfucius

mito94
Membre Relatif
Messages: 165
Enregistré le: 21 Juin 2009, 10:03

par mito94 » 22 Nov 2009, 00:48

laquestion a écrit:c'est la que ça coince...
(ma "solution" ne vous convient pas ?)


pourquoi est ce que sa coince ? on a x>y>z donc x/y > y/z >z/x

exact cela marche quelque soit z différent de 1
en précisant cela je pense que ce que j'ai fait est bon

Doraki
Habitué(e)
Messages: 5021
Enregistré le: 20 Aoû 2008, 13:07

par Doraki » 22 Nov 2009, 01:07

L'équation n'étant pas totalement symétrique (il est invariant par les 3-cycles mais pas par S3 tout entier), l'hypothèse x>=y>=z est importante, (cependant j'ai pas trouvé de contre-exemple dans le cas x<=y<=z).

Sous cette hypothèse, le polynôme symétrique (y-x)(z-y)(x-z) est positif.

En multipliant l'inégalité par xyz et en regroupant tout, il suffit de trouver que

Avatar de l’utilisateur
Olympus
Modérateur
Messages: 1668
Enregistré le: 12 Mai 2009, 13:00

par Olympus » 22 Nov 2009, 01:24

Doraki a écrit:L'équation n'étant pas totalement symétrique (il est invariant par les 3-cycles mais pas par S3 tout entier), l'hypothèse x>=y>=z est importante, (cependant j'ai pas trouvé de contre-exemple dans le cas x<=y<=z).

Sous cette hypothèse, le polynôme symétrique (y-x)(z-y)(x-z) est positif.

En multipliant l'inégalité par xyz et en regroupant tout, il suffit de trouver que


:doh: Encore une solution en quelques lignes de ta part avec la même méthode : soustraction + factorisations de oufs :doh:

 

Retourner vers ⚔ Défis et énigmes

Qui est en ligne

Utilisateurs parcourant ce forum : Aucun utilisateur enregistré et 7 invités

Tu pars déja ?



Fais toi aider gratuitement sur Maths-forum !

Créé un compte en 1 minute et pose ta question dans le forum ;-)
Inscription gratuite

Identification

Pas encore inscrit ?

Ou identifiez-vous :

Inscription gratuite